You are on page 1of 13

26

Kirchhoff 's Laws


Kirchhoff 's laws are two simple laws relating the way that current and voltage behave
in an electrical circuit. Kirchhoff 's current law is concerned with the action of
currents at a point or junction, whilst Kirchhoff 's voltage law looks at the
relationship between the sources of emf and the voltage drops around a closed loop.
You have already used Kirchhoff 's current and voltage laws (without knowing them)
in conjunction with Ohms law to solve series and parallel resistor networks. However
Kirchhoff 's laws allow you to solve circuit problems without having to simplify the
circuit into its equivalent resistances. Kirchhoff 's laws are also particularly useful
where the circuit has two or more supplies.

1. Kirchhoff 's Current Law.


This is stated as follows:
In any circuit the algebraic sum of currents at a point is zero.
ie. The current entering a junction is equal to the current leaving the junction.

Note. The term 'algebraic' means that the direction of the current must be taken into
account.
In the diagram below let all the currents entering the junction be +ve. and all the
currents leaving the junction be -ve.
I2
I1

I3
I1 + (- I2) + (- I3) = 0

Or I1 = I2 + I3 Amperes

2. Kirchhoff 's Voltage law.


This is stated as follows:
In a closed loop the algebraic sum of the product of current and resistance in each
part of the circuit is equal to the resultant emf in the loop.
ie. The sum of the voltage drops around a closed loop is equal to the resultant
emf.

R1 R2
I R1 _ + I R2 _
+
Resultant emf E = I R1 + I R2 Volts

+ E _
27

Worked example
Determine the currents in each part of the circuit shown below using Kirchhoff 's
laws. Then calculate the pd across resistor R2 and the power dissipated in resistor R3.

I3 R3 R4
X 18  2

R1 R2 loop 2
I1 I2
10  5
V2

loop 1

E=6V

Step 1. Insert the currents in each branch of the circuit and indicate their assumed
direction.

Step 2. Write down, using Kirchhoff 's current law the current equation linking
currents I1, I2 and I3 at a point in the circuit. In this case use point ' X '.

Current equation I1 = I2 + I3

Step 3. Select two closed loops in order to produce the voltage equations and select a
direction to work around the loop. Where a source of emf is included in the
loop start at the positive terminal of the source and finish at the negative
terminal.

Step 4.Write down, using Kirchhoff 's voltage law the voltage equations for loop1 and
loop 2.

In loop1 The source of emf is E. The voltage drops are I1R1 and I2R2. These
voltage drops are both positive since we are working around the loop in the
same direction as currents I1 and I2.

Loop 1 - Voltage equation E - I1R1 - I2R2 = 0

Substituting numbers, 6 - 10I1 - 5I2 = 0 ......... (1)

In loop2 There is NO source of emf, therefore 'E' is zero. The voltage drops
working clockwise around the loop from point 'X' are;

I3R3, I3R4 and -I2R2.

The voltage drops I3R3, I3R4 are both positive since we are working around
the loop in the same direction as current I3. Whilst voltage drop I2R2 is
28

negative since we are working around the loop in the opposite direction to
current I2.

Loop 2 - Voltage equation - I3R3 - I3R4 + I2R2 = 0

Substituting numbers, - 18I3 - 2I3 + 5I2 = 0

Therefore, - 20I3 + 5I2 = 0 .........(2)

Step 5. Use the rules of simultaneous equations to obtain the values of the three
currents as follows:
6 - 10I1 - 5I2 = 0 ......... (1)
- 20I3 + 5I2 = 0 ........... (2)

Current equation I1 = I2 + I3

Substitute I1 = I2 + I3 into equation (1) to obtain a second voltage equation in


terms of currents I2 and I3.
6 - 10( I2 + I3 ) - 5I2 = 0
or 6 - 10I2 - 10I3 - 5I2 = 0

therefore 6 - 15I2 - 10I3 = 0 ......... (3)


and rearranging (2) gives 5I2 - 20I3 = 0 ........... (2)

Multiply equation (3) by 2 in order to produce the same number of I3's in


each equation.
12 - 30I2 - 20I3 = 0 ........ (3)
5I2 - 20I3 = 0 ........... (2)

Subtract equation (2) from equation (3).

12 - 35I2 = 0
Therefore I2 = 12 = 0.343 A
35
Substitute the value of I2 into one of the voltage equations, say equation (2).
(5 x 0.343) - 20I3 = 0 ........... (2)

1.714 - 20I3 = 0

Therefore I3 = 1.714 = 0.086 A or 86 mA


20
Use the current equation to find the value of current I1

Since I1 = I2 + I3 = 0.343 + 0.086


I1 = 0.429 A
Therefore:
The pd across resistor R2 is V2 = I2.R2 = 0.343 x 5 = 1.714V
The power dissipated in resistor R3 is P3 = I32.R3 = 0.0862 x 18 = 0.133 W
29

Self Assessment Questions

Note: In ALL Kirchhoff’s problems the circuit diagram showing the assumed
current directions and the voltage loop directions are an integral part of your
solution and MUST be shown. Without these your equations cannot be verified.

1. In the figure shown below determine the currents in each part of the circuit
using Kirchhoff 's laws.

I1
I2 I3
7

8

8
R1 R2 R4
loop 2

E
9

50 V R3
loop 1

Answers: I1 = 2.5A, I2 = 1.25A, I3 = 1.25A


30

Self Assessment Questions (continued)

I1 I2

2
I3

1
R1 R2

10 
R3

E1 E2
12 V 9V

2. Redraw the circuit shown above, and use Kirchhoff's laws to calculate the
values and directions of currents I1, I2, and I3.
[Ans. I1=1.69A, I2= -0.66A and I3=1.03A]

3. A dc generator with an armature resistance of 1 and emf 125V is connected in


parallel with a bank of batteries having an emf of 120V and internal resistance
0.8. A 10 load is connected across their terminals as shown below.
Using Kirchhoff 's laws determine a) the current supplied by the generator and by
the batteries to the load, and b) the pd across the load resistance.

I1

R1 I3 R2
1.0  R3 0.8 
10
E1 E2
G
125V 120V

[Ans 8A and 3.72A respectively, 117.2V]


31

Self Assessment Questions (continued)


I1 I2 I1 I2

2

2
1 I3 I3

3
R1 R2 R1 R2

10 

12 
R3 R3

E1 E2 E1 E2
12 V 9V 24 V 24 V

Fig.3 Fig.4

4. Redraw the circuit shown in fig.3 above, and use Kirchhoff 's laws to calculate
the values and directions of currents I1, I2, and I3.
[Ans. I1=7.31A, I2= 6.84A and I3= -0.47A]

5. Redraw the circuit shown in fig.4 above, and use Kirchhoff 's laws to calculate
a) the values currents I1, I2, and I3, b) the pd across resistor R3 and
c) the power dissipated in resistor R3.
[Ans. I1 = 9.45A, I2 = 9.82A, I3 = - 0.363A, V3 = -4.36V and P3 = 1.59W]

Worked Example - A three supply network when all the emfs are in parallel.
For the three supply circuit shown below, use Kirchhoff 's laws to determine:
a) The values of the currents I1, I2, and I3.
b) The pd across resistor R4 and its polarity
c) The total power dissipated in the internal resistance of the batteries.

I1 A R4 I3
10 
I2
2
3
4

R1 R2 R3

E1 E2 E3
25 V 22 V 24 V
loop 1 loop 2

Assume the current directions shown.

Therefore at point 'A', I2 = I1 + I3

Apply voltage law to each loop, working from the highest potential to the lowest
potential.

In loop 1 Resultant emf E1 - I1R1 - I2R2 - E2 = 0


25 - 4I1 - 3I2 - 22 = 0
3 - 4I1 - 3I2 = 0 ..............(1)
32

In loop 2 Resultant emf E2 + I2R2 + I3R4 + I3R3 - E3 = 0


22 + 3I2 + 10I3 + 2I3 – 24 = 0
- 2 + 3I2 + 12I3 = 0 ..............(2)

Substitute for I2 in equations (1) & (2)

3 - 4I1 - 3(I1 + I3) = 0


3 - 7I1 - 3I3 = 0..................(3)

- 2 + 3(I1 + I3) + 12I3 = 0


- 2 + 3I1 + 15I3 = 0.................(4)

multiply eqn (3) by 5

15 - 35I1 - 15I3 = 0............(5)


eqn(5) + eqn(4)

13
13 - 32I1 = 0 therefore I1   0. 406 A
32

Substitute value of I1 in equation (3)

3 - (7 x 0.406) - 3I3 = 0

0.158
3 - 2.842 = 3I3, therefore I 3   0.052 A
3

Since I2 = I1 + I3, then I2 = 0.406 + 0.052 = 0.458 A

b) Let the pd across resistor R4 be V4 = I3.R4 = 0.052 x 10 = 0.52 V


The end of resistor R4 closest to point 'A' is negative.

c) The total power dissipated in the internal resistance's of the three supplies

PT  I 12 .R1  I 22 .R2  I 32 .R3 Watts


PT  (0.406 2  4)  (0.458 2  3)  (0.052 2  2)
PT  1.294 W
33

Worked Example - A three supply network when 1 of the emf 's is reversed.
Repeat previous worked example but with the polarity of emf E2 reversed.

Leave the current directions as before…

Therefore at point 'A' I2 = I1 + I3


A R4 I3
I1
10 
I2

2
3
4

R1 R2 R3

E1 E2 E3
25 V 22 V 24 V
loop 1 loop 2

With the polarity of E2 reversed, then emf 's E1 & E2 are in series, as are emf 's
E2 & E3. Therefore the emf 's are additive around each loop.

Apply voltage law to each loop, working from the highest potential to the lowest
potential.

In loop 1 Resultant emf E1 - I1R1 - I2R2 + E2 = 0


25 - 4I1 - 3I2 + 22 = 0
47 - 4I1 - 3I2 = 0 ..............(1)

In loop 2 Resultant emf - E2 + I2R2 + I3R4 + I3R3 - E3 = 0


- 22 + 3I2 + 10I3 + 2I3 – 24 = 0
- 46 + 3I2 + 12I3 = 0 ..............(2)

Substitute for I2 in equations (1) & (2)

47 - 4I1 - 3(I1 + I3) = 0


47 - 7I1 - 3I3 = 0 ..................(3)

- 46 + 3(I1 + I3) + 12I3 = 0


- 46 + 3I1 + 15I3 = 0 .................(4)

multiply eqn (3) by 5

235 - 35I1 - 15I3 = 0 ............(5)

eqn(5) + eqn(4)
189
189 - 32I1 = 0 therefore I 1   5. 91 A
32
34

Substitute value of I1 in equation (3)

47 - (7 x 5.91) - 3I3 = 0

5.63
47 - 41.37 = 3I3, therefore I 3   1.88 A
3

Since I2 = I1 + I3, then I2 = 5.91 + 1.88 = 7.8 A

b) Let the pd across resistor R4 be V4 = I3.R4 = 1.88 x 10 = 18.8 V


The end of resistor R4 closest to point 'A' is negative.

c) The total power dissipated in the internal resistances of the three supplies

PT  I 12 .R1  I 22 .R2  I 32 .R3 Watts


PT  (5.912  4)  (7.8 2  3)  (1.88 2  2)
PT  329.3W
35

Self Assessment Questions

1. Determine the value of I3 in the network below.

But there are 3 loops !

The way to tackle this problem is to follow exactly the same procedure but you will
end up with 3 equations ….

R1 I1 I2 R2 R3 I3
5 12 4
I1 - I2 I2 – I3
1 2 3
E1 R4 R5 E2
12V 10 16 10V

loop 1

12 – 5I1 – 10(I1 – I2) = 0 (1)

loop 2

10(I1-I2) – 12 I2 – 16(I2 – I3) = 0 (2)

loop 3

16 (I2 – I3) – 4 I3 – 10 = 0 (3)

… combine equations (1) and (2) to eliminate I1 , then combine resultant with
equation (3) to eliminate I2. From this procedure, you will be left with a value for I3.

(If need be, you could easily plug back the calculated value of I3 into the previous
equation, and so on, to find the other two currents).

Answer : I3 = - 0.5 A
36

Self Assessment Questions (continued)

2. For the circuit shown below determine a) The values of currents I1 , I2 & I3
b) State which batteries are charging or discharging and c) The total power
dissipated in the circuit resistances

[a) 3A , 1A , -2A ; b) discharging , charging , charging ; c) 28W]

I1 I3
I2

1
6
2

R1 R2 R3

E1 E2 E3
18 V 6V 10 V

3. Repeat question 2 above but with the polarity of the 6V supply reversed.

[a) 3.6A , 2.8A , -0.8A ; b) discharging , discharging , charging ; c) 73.6W]

4. For the resistor network shown below apply Kirchhoff’s laws to the following
loops and derive the loop equations in terms of currents I1, I2 and I3 and emf’s
E1, E2 and E3 where applicable.
i) ABCDGHIA
ii) CDEFGHC
iii) EDCBAIHGFE

B R2 I1 C R4 I2 D R6 I3 E

R1
R3 R5 E3
A
E

I H E2 G

Answers
i) E1 + E2 = I1(R1 + R2) + I2(R4 + R5) + I3R5
ii) E2 - E3 = -I1R3 + I2(R3 + R4) - I3R6
iii) E1 + E2 – E3 = I1(R1 + R2) + I2R4 - I3R6
37

Self Assessment Questions (continued)

5. For the circuit in question 4 (previous page), determine the values of currents I1, I2
and I3, if the resistance values are: R1 = 2, R2 =R4 = R6 = 8 and R3 = R5 = 4,
and the values of the emf’s are E1 =15V, E2 = 10V and E3 = 20V.
[Ans. 1.232A, 0.563A, 1.48A]

6. For the circuit shown below,


a) Show that the magnitudes of currents IA and IB are 65/71 A and -14/71 A
respectively
b) Determine the value of current IR1 [Ans. 0.718A]
c) Obtain the terminal voltages of cells A & B
[Ans. 8.17V & 3.59V respectively]
d) Calculate the energy used in resistor R2 in 30 minutes [Ans. 7543 J]

2 EA = 10V
Cell A

I
IR1 R1
5 5
I

Cell B
3 EB = 3V
38

Self Assessment Questions (continued)

7. Three batteries having emf’s of 10V, 12V & E respectively and negligible internal
resistance are connected as shown below.
a) With the switch S open calculate the magnitude and direction of the current in
each part of the circuit. [Ans. 0.667A from ‘c’ to ‘a’ , 0.667A from ‘a’ to ‘d’]
b)Given that the current in branch ‘b’ to ‘c’ is 4A, flowing towards ‘c’ when the
switch is closed, calculate the magnitude of E and the currents in each part of
the circuit under this condition.
[Ans. 8V, 2A from ‘c’ to ‘a’, 2A from ‘d’ to a’]

a b

1 2 4

E
12V
10V S

d c

You might also like